Inequality by Kristian Vasilev

Algebra Level 4


The answer is 36.

This section requires Javascript.
You are seeing this because something didn't load right. We suggest you, (a) try refreshing the page, (b) enabling javascript if it is disabled on your browser and, finally, (c) loading the non-javascript version of this page . We're sorry about the hassle.

3 solutions

Martin Nikolov
Nov 19, 2014

L e t B = 6 ( a c + b a + c b ) a n d C = a 2 + 9 c + b 2 + 9 a + c 2 + 9 b A = B + C U s i n g t h e i n e q u a l i t y ( a c + b a + c b ) 3 w e g e t B 18 U s i n g t h e i n e q u a l i t y x 2 + y 2 2 x y w e g e t C 6 ( a c + b a + c b ) = B C 18 A 36 Let\quad B=6\left( \frac { a }{ c } +\frac { b }{ a } +\frac { c }{ b } \right) \quad and\quad C=\frac { { a }^{ 2 }+9 }{ c } +\frac { { b }^{ 2 }+9 }{ a } +\frac { { c }^{ 2 }+9 }{ b } \\ \Rightarrow A=B+C\\ Using\quad the\quad inequality\quad \left( \frac { a }{ c } +\frac { b }{ a } +\frac { c }{ b } \right) \ge 3\quad we\quad get\quad B\ge 18\\ Using\quad the\quad inequality\quad { x }^{ 2 }+{ y }^{ 2 }\ge 2xy\quad we\quad get\quad C\ge 6\left( \frac { a }{ c } +\frac { b }{ a } +\frac { c }{ b } \right) =B\Rightarrow C\ge 18\\ \Rightarrow A\ge 36

With equality case a = b = c = 3 a=b=c=3 .

Daniel Liu - 6 years, 5 months ago

Log in to reply

Thanks, I forgot to write it :-)

Martin Nikolov - 6 years, 5 months ago
Michael Ng
Dec 25, 2014

By the Cauchy-Schwarz Inequality, we have: Σ ( a + 3 ) 2 c ( 9 + a + b + c ) 2 a + b + c \Sigma \frac{(a+3)^2}{c} \geq \frac{(9+a+b+c)^2}{a+b+c} Now by the AM-GM inequality: ( 9 + a + b + c ) 2 36 ( a + b + c ) (9+a+b+c)^2 \geq 36(a+b+c) This gives us a minimum of 36 \boxed{36} which is achieved when a , b , c = 3 a,b,c=3 .

Nice solution .

Kristian Vasilev - 6 years, 5 months ago

Ahh same solution. I belive this particular version of Cauchy-Schwarz is called Titu's Lemma.

A Former Brilliant Member - 6 years, 5 months ago
Thinula De SIlva
Jan 3, 2015

By Cauchy-Schwarz's Inequality, we have Σ ( a + 3 ) 2 c ( a + b + c + 9 ) 2 a + b + c \Sigma \frac{(a+3)^2}{c} \ge \frac{(a+b+c+9)^2}{a+b+c} We have that ( a + b + c + 9 ) 2 = ( a + b + c ) 2 + 18 ( a + b + c ) + 81 (a+b+c+9)^2=(a+b+c)^2+18(a+b+c)+81 . Thus, we want to minimize ( a + b + c ) 2 + 18 ( a + b + c ) + 81 a + b + c \frac{(a+b+c)^2+18(a+b+c)+81}{a+b+c} = ( a + b + c ) + 81 a + b + c + 18 =(a+b+c)+\frac{81}{a+b+c}+18 By AM-GM, we have ( a + b + c ) + 81 a + b + c 18 (a+b+c)+\frac{81}{a+b+c} \ge 18 Thus, the minimum value of 36 \boxed{36} , when a , b , c = 3 a,b,c=3

0 pending reports

×

Problem Loading...

Note Loading...

Set Loading...